Produit infini

etanche
Modifié (November 2021) dans Analyse
Bonjour
Montrer pour $|q|<1$
$$\prod_{n=1}^{\infty} (1-q^n)(1-q^{6n-1})(1-q^{6n-5}) =2\sum_{n=0}^{\infty}  \cos((2n+1)\frac{\pi}{3} ) q^{\frac{n(n+1)}{2}}.$$ Merci.

Réponses

  • LOU16
    Modifié (November 2021)
    Bonjour
    Cette identité me semble suspecte: je ne parviens pas en effet à trouver une erreur dans ce qui suit.
    $a_n := 2\cos\Big((2n+1)\pi/3\Big).$  L'égalité proposée est équivalente à:
    $$  \sum_{n=0}^{+\infty}a_n q^{n(n+1)/2}\prod _{n=1}^{+\infty} (1-q^{6n}) = \prod _{n=1}^{+\infty}(1-q^n) \left(\prod _{n=1}^{+\infty}(1-q^{6n}) (1-q^{6n-5})(1-q^{6n-1}) \right),$$
    ce qui, avec la "formule du triple produit de Jacobi" et ses conséquences, s'écrit aussi :
    $$  \sum_{n=0}^{+\infty}a_n q^{n(n+1)/2}\sum_{n\in \Z} (-1)^nq^{3n(3n+1)}= \sum_{n\in \Z}(-1)^n q^{n(3n+1)/2}\sum_{n\in \Z}(-1)^n q^{3n^2-2n}.$$
    Or le coefficient de $q^{23}$ est nul dans le membre de gauche, et vaut $-1$ dans le membre de droite.
  • etanche
    Modifié (November 2021)
    Dans l’identité initiale le coefficient de $q^{23}$  est nul des deux côtés me semble-t-il.
  • Bonjour,

    La relation donnée me semble fausse. 

    On sait par coeur ou on trouve dans le tables des fonctions thêta elliptiques $\displaystyle v_2(z,e^{-\pi \lambda}) = \sum_{n \in \Z} e^{- \pi \lambda (n+1/2)^2} \cos((2n+1) z) = 2 e^{- \pi \lambda/4} \cos z \prod_{n \geq 1} (1-e^{-2 \pi \lambda n}) (1+2 \cos (2z) e^{-2 \pi \lambda n}+e^{-4 \pi \lambda n})$ avec des notations à deviner.

    Cette formule, appliquée avec $\displaystyle z=2 \pi/3, q=e^{- 2 \pi \lambda}$, et avec la somme sur $\Z$ transformée en somme sur $\N$ par un changement d'indice, donne $\displaystyle 2 \sum_{n \geq 0} q^{n(n+1)/2} \cos((2n+1) \pi/3) = \prod_{n \geq 1} (1-q^n)(1-q^n+q^{2n}).$
  • Et on a $\prod_{n=1}^{\infty}(1-q^n + q^{2n}) = \prod_{n=1}^{\infty}(1-q^{6n-5})(1-q^{6n-1})=\prod_{n=0}^{\infty}(1-q^{6n+5})(1-q^{6n+1})$ 
    voir le post de Favst
     https://math.stackexchange.com/questions/3733360/an-application-of-integer-partitions?noredirect=1
  • LOU16
    Modifié (November 2021)
    Bonsoir,
     J'ai en effet commis une erreur en oubliant un terme dans l'obtention du coefficient de $q^{23}$ de "mon" second membre, et ce coefficient est bien égal à $0$ comme attendu. Désolé.
    Après l'avoir interrompue à la suite de cette erreur, j'ai donc repris la recherche d'une justification de cette identité, jusqu'à en atteindre une preuve combinatoire assez compliquée , que je vais m'efforcer de simplifier avant de la rédiger
  • etanche
    Modifié (November 2021)
    Tentative d’une preuve taupinale 

    Notons G(q) l’expression de gauche, D(q) l’expression de droite 
    Prendre le logarithme montrer que $F(q)=\ln(G(q)) - \ln(D(q))$ a une dérivée nulle, puis $G(q)=\lambda D(q)$ 
    pour trouver $\lambda$ faire $q=0$. 
  • LOU16
    Modifié (November 2021)
    Bonjour
    Pour tout entier $n, \:\chi(n)$ est défini par: $ \:\: \chi (n) \in \{0,1,-1\}, \: \chi(n) \equiv n \mod 3.\quad$  Alors
    $\:\forall n \in \N,\quad 2\cos [(2n+1)\pi/3] = -\chi(n) -\chi(-n-1),\quad \displaystyle \sum_{n\in\N} 2\cos [(2n+1)\pi/3]q^{n(n+1)/2} =\sum_{n\in\Z}-\chi(n)q^{n(n+1)/2}.$
     $E(q) :=\displaystyle\sum_{n\in\Z}-\chi(n)q^{n(n+1)/2}\displaystyle\sum_{n\in\Z}(-1)^nq^{3n(3n+1)}:= \sum_{n\in \N}e_nq^n, \qquad F(q):=\displaystyle\sum_{n\in\Z}(-1)^nq^{n(3n+1)/2}\displaystyle\sum_{n\in\Z}(-1)^nq^{3n^2-2n} :=\sum_{n\in \N}f_nq^n .$
    Comme je l'ai écrit dans un message précédent, l'identité initiale est équivalente à $\:E(q) =F(q)$, c'est-à-dire à $\:\: \boxed{\forall n \in \N, \:\: e_n =f_n.}$
    Quelques notations.
    $\forall n \in \N, \:\:\:\mathcal E_n :=\Big\{(x,y)\in \Z^2\mid \dfrac {x(x+1)}2+ 3y(3y+1)=n\Big\}\quad \mathcal F_n :=\Big\{(z,t)\in \Z^2\mid \dfrac {z(3z+1)}2+ 3t^2-2t=n\}\Big\}$
    $\forall (i,j) \in \{0,1,-1\}\times\{0,1\}, \:\: \mathcal E_n^{i,j} := \Big\{(x,y) \in \mathcal E_n \mid x\equiv i \mod3, \: y\equiv j \mod 2 \Big \}, $
    $\mathcal F_n^{i,j} := \Big\{(z,t) \in \mathcal F_n \mid t-z\equiv i \mod3, \: t-z\equiv j \mod 2\Big \}, \quad E_n^{i,j}: =\#\mathcal E_n^{i,j}, \:\:F_n^{i,j} :=\#\mathcal F_n^{i,j}.$
    Alors : $\forall n \in \N, \:\: e_n =-E_n^{1,0} +E_n^{1,1}+E_n^{-1,0}-E_n^{-1,1}, \quad f_n= F_n^{0,0}-F_n^{0,1}+F_n^{1,0}-F_n^{0,1}+F_n^{-1,0}-F_n^{-1,1}.$
    Il reste donc à prouver que :
    $$\boxed{ \forall n \in \N, \quad -E_n^{1,0} +E_n^{1,1}+E_n^{-1,0}-E_n^{-1,1}= F_n^{0,0}-F_n^{0,1}+F_n^{1,0}-F_n^{1,1}+F_n^{-1,0}-F_n^{-1,1}. \quad (\bigstar)}$$
    En scrutant les connexions entre les $ \mathcal E_n^{i,j}$ et les $ \mathcal F_n^{i,j}, \:$ on parvient à déceler l'existence de transformations  $\Phi, \Psi, \Theta$ dotées des propriétés suivantes, faciles à vérifier :
    $\bullet \quad \Phi: (x,y) \mapsto \left( \dfrac {x-1-6y}3, \dfrac {x+2+3y}3 \right) $ réalise une bijection de $\:\mathcal E_n^{1,0}\: $ sur $\: \mathcal F_n^{1,1}\:$ et une bijection de $\:\mathcal E_n^{1,1}\: $ sur $\: \mathcal F_n^{1,0}.$
    $\bullet \quad \Psi: (x,y) \mapsto \left( \dfrac {x+1+6y}3, \dfrac {x+1-3y}3 \right) $ réalise une bijection de $\:\mathcal E_n^{-1,0}\: $ sur $\: \mathcal F_n^{0,0}\: $ et une bijection de $\:\mathcal E_n^{-1,1}\: $ sur $\: \mathcal F_n^{0,1}.$
    $\bullet \quad \Theta: (z,t)\mapsto \left( \dfrac{-z+4t -2}3, \dfrac{2z+t+1}3 \right)\:\: $ est une involution qui échange $\mathcal F_n^{-1,0}$ et $\mathcal F_n^{-1,1}.$
    Il résulte de ces observations les relations
    $ E_n^{1,0}= F_n^{1,1}, \:\: E_n^{1,1}= F_n^{1,0}, \:\: E_n^{-1,0}= F_n^{0,0}, \:\: E_n^{-1,1}= F_n^{0,1}, \:\: F_n^{-1,0}= F_n^{-1,1},\:$ qui assurent la validité de $(\bigstar).$
  • etanche
    Modifié (November 2021)
    Pour les fonctions thêta : Hardy and Wright Introduction to the theory of numbers.
  • Boécien
    Modifié (November 2021)
    Bravo LOU16...Il me semble que les coefficients sont donnés par cette suite dans l'OEIS où il y a ce commentaire : "This is an example of the quintuple product identity in the form f(a*b^4, a^2/b) - (a/b) * f(a^4*b, b^2/a) = f(-a*b, -a^2*b^2) * f(-a/b, -b^2) / f(a, b) where a = x^2, b = x" !!!
Connectez-vous ou Inscrivez-vous pour répondre.